Difference between revisions of "2006 AMC 12A Problems/Problem 4"

(Solution)
(added category and link to previous and next problem)
Line 13: Line 13:
 
== See also ==
 
== See also ==
 
* [[2006 AMC 12A Problems]]
 
* [[2006 AMC 12A Problems]]
 +
*[[2006 AMC 12A Problems/Problem 3|Previous Problem]]
 +
*[[2006 AMC 12A Problems/Problem 5|Next Problem]]
 +
 +
[[Category:Introductory Number Theory Problems]]

Revision as of 18:46, 5 November 2006

Problem

A digital watch displays hours and minutes with AM and PM. What is the largest possible sum of the digits in the display?

$\mathrm{(A) \ } 17\qquad \mathrm{(B) \ } 19\qquad \mathrm{(C) \ } 21\qquad \mathrm{(D) \ } 22$

$\mathrm{(E) \ }  23$

Solution

The sum of digits is largest when the number of hours is $9$ and the number of minutes is $59$. Therefore, the largest possible sum of digits is $9+5+9=23$. The answer is E.

See also